Solving an Initial Value Problem with a Matrix and a Given Vector: A Case Study

  • Thread starter Thread starter Shackleford
  • Start date Start date
  • Tags Tags
    Ivp
Click For Summary
The discussion focuses on solving the initial value problem x' = Ax with the given vector v = [1,2]^T and matrix A = [(1, -1)^T (1, -1)^T]. The approach taken involves calculating e^(tA)v = v[I + tA], leading to the solution x(t) = e^(tA)v. The hint provided, A^2v = 0, suggests that using the matrix exponential simplifies the process without needing to find a fundamental set of solutions. One participant expresses confusion about not following the general procedure and reflects on receiving a low score on a related exam question due to errors in their calculations. The conversation concludes with a plan to contact the professor for clarification and potential re-evaluation of the exam work.
Shackleford
Messages
1,649
Reaction score
2
x' = Ax, x(0) = v,

where A is the matrix in problem 6 and v = [1,2]^T. Do not use the eigenvalues and the eigenvectors of A. (Hint A^2v = 0).A = [ (1, -1)^T (1, -1)^T ]

All I did was calculate e^(tA)v = v[I + tA].

In this case, an IVP, x(t) is the solution. x(t) = e^(tA)v.

Since I wasn't asked to find a fundamental set of solutions, I didn't need to calculate e^(tA)v for every v that's a basis in R^n, right?
 
Last edited:
Physics news on Phys.org
Your solution looks good to me, and working with the exponential was the way to go. That's the reason for hint, I'm pretty sure.

e^(tA) = I + tA =
[t+1 t]
[-t -t+1]
solves the differential equation, and x(0) = v, so all is good.
 
Mark44 said:
Your solution looks good to me, and working with the exponential was the way to go. That's the reason for hint, I'm pretty sure.

e^(tA) = I + tA =
[t+1 t]
[-t -t+1]
solves the differential equation, and x(0) = v, so all is good.

Yeah. I figured that. I guess I'm a little fuzzy on why you don't have to use the general procedure. They have a little proof in the book which is fairly straightforward. I understand that.
 
Well, I got my exam back today after the 2.5-hour long final.

I only got 5/15 for that work. Apparently, and rightfully so, I had the v on the wrong side. I Also forgot to to put the v in tAv.

I want to email the professor for points on that problem and another one. Are you sure I did it correctly? I'll probably scan the questions and my work to see if I should ask for more points.
 
Question: A clock's minute hand has length 4 and its hour hand has length 3. What is the distance between the tips at the moment when it is increasing most rapidly?(Putnam Exam Question) Answer: Making assumption that both the hands moves at constant angular velocities, the answer is ## \sqrt{7} .## But don't you think this assumption is somewhat doubtful and wrong?

Similar threads

  • · Replies 2 ·
Replies
2
Views
2K
Replies
2
Views
2K
  • · Replies 2 ·
Replies
2
Views
1K
Replies
5
Views
2K
  • · Replies 3 ·
Replies
3
Views
2K
  • · Replies 4 ·
Replies
4
Views
1K
  • · Replies 24 ·
Replies
24
Views
3K
  • · Replies 6 ·
Replies
6
Views
1K
Replies
8
Views
3K
  • · Replies 11 ·
Replies
11
Views
3K